Questions tagged [independence-results]

This tag is for questions about proving that some statement is independent from a theory, meaning it is neither provable nor refutable from that theory. Common examples are the continuum hypothesis from the axioms of ZFC, and the axiom of choice from the axioms of ZF.

14 questions with no upvoted or accepted answers
Filter by
Sorted by
Tagged with
13 votes
0 answers
315 views

$\mathsf{AC}_\mathsf{WO}+\mathsf{AC}^\mathsf{WO}\Rightarrow \mathsf{AC}$?

Let $\mathsf{AC}_\mathsf{WO}$: Every well-orderable family of non-empty sets has a choice function. $\mathsf{AC}^\mathsf{WO}$: Every family of non-empty well-orderable sets has a choice function. My ...
Lorenzo's user avatar
  • 2,134
11 votes
0 answers
242 views

Existence of a strong antichain

Call an antichain (set of pairwise incomparable elements) $A$ of a poset $P$ strong if for every $p,q \in P$ with $p \leq q$ there exists an $a\in A$ which is comparable with both $p$ and $q$. ...
Attila Joó's user avatar
8 votes
0 answers
224 views

Metrically Ramsey ultrafilters

On Thuesday I was in Kyiv and discussed with Igor Protasov the system of MathOverflow and its power in answering mathematical problems. After this discussion Igor Protasov suggested to ask on MO the ...
Taras Banakh's user avatar
  • 40.9k
7 votes
0 answers
172 views

Asymptotically discrete ultrafilters

Definition 1. A ultrafilter $\mathcal U$ on $\omega$ is called discrete (resp. nowhere dense) if for any injective map $f:\omega\to \mathbb R$ there is a set $U\in\mathcal U$ whose image $f(U)$ is a ...
Taras Banakh's user avatar
  • 40.9k
6 votes
0 answers
366 views

How bad a proper forcing of size $\aleph_1$ can be?

This question concerns proper forcings of size $\aleph_1$. In the context of $\rm ZFC+\neg CH$, I couldn't find any counter example to the following property. Suppose $\mathbb P$ is a proper forcing ...
Rahman. M's user avatar
  • 2,341
5 votes
0 answers
206 views

Questions about very fat sets

If $\kappa$ is a regular uncountable cardinal, we call a set $S\subseteq\kappa$ fat if for every $\alpha<\kappa$ and every club $C\subseteq\kappa$, there is a closed subset of $S\cap C$ of ...
Hannes Jakob's user avatar
  • 1,612
5 votes
0 answers
171 views

Selecting an almost disjoint family in a given family of sets

A family $\mathcal A$ of infinite subsets of $\omega$ is called almost disjoint if for any distinct sets $A,B\in\mathcal A$ the intersection $A\cap B$ is finite. Let $\mathfrak a'$ be the largest ...
Taras Banakh's user avatar
  • 40.9k
3 votes
0 answers
198 views

Independence and truth in PA

By $\textbf{PA}$ I will mean the usual first-order Peano Arithmetic. I will denote an element of $\mathbb{N}$ by $n$, and by $[n]$ I will denote the corresponding term in the language of $\textbf{PA}$:...
jg1896's user avatar
  • 2,733
3 votes
0 answers
146 views

The "absolute" version of the Axiom Schema of Replacement in ZFC

The well-known Axiom Schema of Replacement in ZFC says that for any formula $\varphi$ of the Set Theory with free variables among $w_1,\dots,w_n,A,x,y$ the following holds: $$\forall w_1,\dots,w_n\;\...
Taras Banakh's user avatar
  • 40.9k
3 votes
0 answers
77 views

Is $\mathfrak q_0$ equal to the smallest cardinality of a second-countable $T_1$-space which is not a $Q$-space?

A topological space $X$ is a $Q$-space if every subset of $X$ is of type $G_\delta$. The smallest cardinality of a metrizable separable space which is not a $Q$-space is denoted by $\mathfrak q_0$ and ...
Taras Banakh's user avatar
  • 40.9k
3 votes
0 answers
122 views

The existence of $T$-ultrafilters in ZFC

Looking at this MO-problem, my collegue Igor Protasov suggested to ask on Mathoverflow his old question on $T$-ultrafilters hoping that somebody on MO can solve it. First I recall the necessary ...
Taras Banakh's user avatar
  • 40.9k
2 votes
0 answers
120 views

Two small uncountable cardinals related to Q-sets

A subset $A$ of the real line is called a Q-set if any subset of of $A$ is of type $F_\sigma$ in $A$. Let $\mathfrak q_0$ be the smallest cardinality of a subset $X\subset\mathbb R$ which is not a Q-...
Taras Banakh's user avatar
  • 40.9k
1 vote
0 answers
261 views

About Whitehead's problem

Hi I am new to proofs of consistency and independence with ZFC of some claims. I have read "The uses of set theory" by Judith Roitman, in that article it is mentioned that the Whitehead ...
Gabriel Medina's user avatar
0 votes
0 answers
180 views

How are incompleteness and independence proofs related?

(1) Some typical inclompleteness proofs use a kind of fixed point argument - for certain $\Phi$ you find a $\varphi$ with $\Phi(\mathrm{code}(\varphi))\leftrightarrow\varphi$. (2) Some independence ...
მამუკა ჯიბლაძე's user avatar